Sei sulla pagina 1di 4

Volume 6, Number 2 April 2001 – May 2001

Olympiad Corner Base n Representations


Kin Y. Li
The 2000 Canadian Mathematical
Olympiad
When we write down a number, it is 1022220 , so A < 22220 × 9 = 199980.
understood that the number is written in Then B < 1 + 9 × 5 = 46 and the sum of
Problem 1. At 12:00 noon, Anne, Beth
base 10. We learn many interesting facts the decimal digits of B is at most 3+9=12.
and Carmen begin running laps around a
at a very young age. Some of these can Now 4444 ≡ 7 (mod 9) and 7 3 = 343
circular track of length three hundred
be easily explained in terms of base 10 ≡ 1 (mod 9) imply 44443 ≡ 1 (mod 9).
meters, all starting from the same point
representation of a number. Here is an Then 4444 4444 = (4444 3 )1481 4444 ≡ 7
on the track. Each jogger maintains a
example. (mod 9). By the remarks above, A, B and
constant speed in one of the two possible
Example 1. Show that a number is the sum of the decimal digits of B also
directions for an indefinite period of time.
divisible by 9 if and only if the sum of its have remainder 7 when divided by 9. So
Show that if Anne’s speed is different
digits is divisible by 9. How about the sum of the decimal digits of B being
from the other two speeds, then at some
divisibility by 11? at most 12 must be 7.
later time Anne will be at least one
Although base 10 representations are
hundred meters from each of the other Solution. Let M = d m 10 m +  + d110
common, numbers expressed in other
runners. (Here, distance is measured + d 0 , where d i = 0, 1, 2, …, 9. The
bases are sometimes useful in solving
along the shorter of the two arcs binomial theorem tells us 10 k = (9 + 1) k
problems, for example, base 2 is common.
separating two runners). = 9 N k + 1. So
Here are a few examples using other
M = d m (9 N m + 1) +  + d1 (9 + 1) + d 0 bases.
Problem 2. A permutation of the
integers 1901, 1902, …, 2000 is a = 9( d m N m +  + d 1 ) + ( d m +  + d 1 + d 0 ).
Example 3. (A Magic Trick) A magician
sequence a1 , a 2 , …, a100 in which Therefore, M is a multiple of 9 if and asks you to look at four cards. On the first
each of those integers appears exactly only if d m +  + d1 + d 0 is a multiple of card are the numbers 1, 3, 5, 7, 9, 11, 13,
once. Given such a permutation, we form 9. 15; on the second card are the numbers 2,
the sequence of partial sums 3, 6, 7, 10, 11, 14, 15; on the third card are
Similarly, we have 10 k = 11N k′ + ( −1) k .
s1 = a1 , s2 = a1 + a 2 , the numbers 4, 5, 6, 7, 12, 13, 14, 15; on
So M is divisible by 11 if and only if
s3 = a1 + a 2 + a 3 , ..., s100 = a1 + a 2 +  + a100 . the fourth card are the numbers 8, 9, 10,
( −1) m d m +  − d1 + d 0 is divisible by
(continued on page 4) 11, 12, 13, 14, 15. He then asks you to
11.
Editors: 張 百 康 (CHEUNG Pak-Hong), Munsang College, HK pick a number you saw in one of these
高 子 眉 (KO Tsz-Mei) Remarks. In fact, we can also see that the
梁 達 榮 (LEUNG Tat-Wing), Appl. Math Dept, HKPU cards and hand him all the cards that have
李 健 賢 (LI Kin-Yin), Math Dept, HKUST
remainder when M is divided by 9 is the
that number on them. Instantly he knows
吳 鏡 波 (NG Keng-Po Roger), ITC, HKPU same as the remainder when the sum of
the number. Why?
Artist: 楊 秀 英 (YEUNG Sau-Ying Camille), MFA, CU the digits of M is divided by 9. Recall the
Acknowledgment: Thanks to Elina Chiu, MATH Dept, notation a ≡ b (mod c) means a and b Solution. For n = 1, 2, 3, 4, the numbers
HKUST for general assistance.
have the same remainder when divided on the n-th card have the common feature
On-line: http://www.math.ust.hk/mathematical_excalibur/ that their n-th digits from the end in base 2
by c. So we have M ≡ d m +  + d1 + d 0
The editors welcome contributions from all teachers and
(mod 9). representation are equal to 1. So you are
students. With your submission, please include your name,
address, school, email, telephone and fax numbers (if handing the base 2 representation of your
available). Electronic submissions, especially in MS Word, The following is an IMO problem that
are encouraged. The deadline for receiving material for the
number to the magician. As the numbers
can be solved using the above remarks.
next issue is June 30, 2001. are less than 2 4 , he gets your number
For individual subscription for the next five issues for the Example 2. (1975 IMO) Let A be the sum
01-02 academic year, send us five stamped self-addressed
easily.
envelopes. Send all correspondence to: of the decimal digits of 44444444 , and B
Dr. Kin-Yin Li
Remarks. A variation of this problem is
be the sum of the decimal digits of A. Find
Department of Mathematics the following. A positive integer less than
Hong Kong University of Science and Technology the sum of the decimal digits of B.
Clear Water Bay, Kowloon, Hong Kong 2 4 is picked at random. What is the least
Fax: 2358-1643 Solution. Since 4444 4444 < (105 ) 4444 = number of yes-no questions you can ask
Email: makyli@ust.hk
Mathematical Excalibur, Vol. 6, No. 2, Apr 01 – May 01 Page 2

that always allow you to know the number? rightmost digit in base 3 where x differs This implies m is not divisible by p. Let
Four questions are enough as you can ask from y, then that digit for z is a 2, a gcd stand for greatest common divisor (or
if each of the four digits of the number in contradiction. highest common factor). Then gcd(m, p)
base 2 is 1 or not. Three questions are not Example 6. Let [r ] be the greatest = 1. Now
enough as there are 15 numbers and three integer less than or equal to r. Solve the (m + cp k ) 2 = m 2 + 2mcp k + c 2 p 2 k
questions can only provide 2 3 = 8 equation = 1 + p +  + p k −1 + ( a + 2mc ) p k + .
different yes-no combinations. [ x ] + [2 x ] + [4 x ] + [8 x ] Since gcd( 2m, p ) = 1, there is a positive
Example 4. (Bachet’s Weight Problem) +[16 x ] + [32 x ] = 12345. integer c such that ( 2m)c ≡ 1 − a (mod p).
Give a set of distinct integral weights that Solution. If x is a solution, then since This implies a + 2mc is of the form
allowed you to measure any object having r − 1 < [ r ] ≤ r, we have 63x – 6 < 12345 1 + Np and so ( m + cp k ) 2 will end in at
weight n = 1, 2, 3,  , 40 on a balance. ≤ 63x. It follows that 195 < x < 196. least (k + 1) 1's as required.
Can you do it with a set of no more than Now write the number x in base 2 as Example 9. Determine which binomial
four distinct integral weights? 11000011.abcde, where the digits n!
coefficients C rn = are odd.
Solution. Since the numbers 1 to 40 in a , b, c, d , e,  are 0 or 1. Substituting r! ( n − r )!
base 2 have at most 6 digits, we can do it this into the equation, we will get 12285 +
Solution. We remark that modulo
with the set 1, 2, 4, 8, 16, 32. To get a set 31a + 15b + 7c + 3d + e = 12345. Then arithmetic may be extended to
with fewer weights, we observe that we 31a + 15b + 7c + 3d + e = 60, which is polynomials with integer coefficients. For
can put weights from this set on both sides impossible as the left side is at most 31 + example, (1 + x ) 2 = 1 + 2 x + x 2 ≡ 1 + x 2
of the balance! Consider the set of 15 + 7 + 3 + 1 = 57. Therefore, the (mod 2). If n = a m +  + a1 , where the
weights 1, 3, 9, 27. For example to equation has no solution. ai ’s are distinct powers of 2. We have
determine an object with weight 2, we can Example 7. (Proposed by Romania for k k

put it with a weight of 1 on one side to (1 + x ) 2 ≡ 1 + x 2 (mod 2) by induction


1985 IMO) Show that the sequence {a n }
balance a weight of 3 on the other side. on k and so
defined by a n = [n 2 ] for n = 1, 2, 3, …
Note the sum of 1, 3, 9, 27 is 40. For any (1 + x ) n ≡ (1 + x a m )(1 + x a1 ) (mod 2).
(where the brackets denote the greatest
integer n between 1 and 40, we can write it integer function) contains an infinite The binomial coefficient C rn is odd if
in base 3. If the digit 2 appears, change it number of integral powers of 2. and only if the coefficient of x in
r

to 3 − 1 so that n can be written as a (1 + x a m )(1 + x a1 ) is 1, which is


unique sum and difference of 1, 3, 9, 27. Solution. Write 2 in base 2 as
equivalent to r being 0 or a sum of one or
For example, 22 = 2 ⋅ 9 + 3 + 1 = (3 – 1)9 b0 .b1b2 b3  , where each bi = 0 or 1.
more of the ai ’s. For example, if
+ 3 + 1 = 27 – 9 + 3 + 1 suggests we put Since 2 is irrational, there are infinitely
n = 21 = 16 + 4 + 1, then C rn is odd for r
the weights of 22 with 9 on one side and many bk = 1. If bk = 1, then in base 2,
= 0, 1, 4, 5, 16, 17, 20, 21 only.
the weights of 27, 3, 1 on the other side. 2 k −1 2 = b0 bk −1.bk  . Let m =
Example 10. (1996 USAMO) Determine
Example 5. (1983 IMO) Can you choose [2 k −1 2 ], then
1983 pairwise distinct nonnegative (with proof) whether there is a subset X of
k −1 k −1 k −1 1
integers less than 105 such that no three 2 2 − 1 < [2 2] = m < 2 2 − . the integers with the following property:
2
are in arithmetic progression? for any integer n there is exactly one
Multiplying by 2 and adding 2 , we
solution of a + 2b = n with a , b ∈ X .
Solution. Start with 0, 1 and at each step k k 2
get 2 < ( m + 1) 2 < 2 + . Then This is a difficult problem. Here we will
add the smallest integer which is not in 2
try to lead the reader to a solution. For a
arithmetic progression with any two [( m + 1) 2 ] = 2 k .
problem that we cannot solve, we can try
preceding terms. We get 0, 1, 3, 4, 9, 10,
12, 13, 27, 18, … . In base 3, this Example 8. (American Mathematical to change it to an easier problem. How
sequence is Monthly, Problem 2486) Let p be an odd about changing the problem to positive
0, 1, 10, 11, 100, 101, 110, 111, 1000, 1001, ... prime number. For any positive integer k, integers, instead of integers? At least we
show that there exists a positive integer m do not have to worry about negative
(Note this sequence is the nonnegative
such that the rightmost k digits of m 2 , integers. That is still not too obvious how
integers in base 2.) Since 1982 in base 2 is to proceed. So can we change it to an even
when expressed in the base p, are all 1's.
11110111110, so switching this from base simpler problem? How about changing 2
3 to base 10, we get the 1983th term of the Solution. We prove by induction on k.
to 10?
sequence is 87843 < 105. To see this For k = 1, take m = 1. Next, suppose
sequence works, suppose x, y, z with x <y m 2 in base p, ends in k 1's, i.e.
m 2 = 1 + p +  + p k −1 + ( ap k + ).
< z are three terms of the sequence in (continued on page 4)

arithmetic progression. Consider the


Mathematical Excalibur, Vol. 6, No. 2, Apr 01 – May 01 Page 3

Problem Corner Problem 121. Prove that any integer


greater than or equal to 7 can be written Let AD, BE and CF be the angle bisectors
We welcome readers to submit their as a sum of two relatively prime integers, of ∆ABC , where D is on BC, E is on CA
solutions to the problems posed below
both greater than 1. and F is on AB. Since ∠ADC = ∠ABD
for publication consideration. Solutions
(Two integers are relative prime if they + ∠BAD > ∠ABD, there is a point K on
should be preceded by the solver’s name,
share no common positive divisor other CA such that ∠ADK = ∠ABD. Then
home (or email) address and school
than 1. For example, 22 and 15 are ∆ABD is similar to ∆ADK . So AB/AD
affiliation. Please send submissions to
relatively prime, and thus 37 = 22 + 15 = AD/AK. Then AD 2 = AB ⋅ AK <
Dr. Kin Y. Li, Department of
represents the number 37 in the desired AB ⋅ CA . Similarly, BE 2 < BC ⋅ AB and
Mathematics, Hong Kong University of
way.) (Source: Second Bay Area CF 2 < CA ⋅ BC. Multiplying these
Science & Technology, Clear Water Bay,
Kowloon. The deadline for submitting Mathematical Olympaid) in-equalities and taking square roots, we
solutions is June 30, 2001. Solution. CHAO Khek Lun Harold (St. get AD ⋅ BE ⋅ CF < AB ⋅ BC ⋅ CA .
Paul’s College, Form 6), CHIU Yik Yin
(St. Joseph’s Anglo-Chinese School, Other commended solvers: CHAO
Problem 126. Prove that every integer
Form 5), CHONG Fan Fei (Queen’s Khek Lun Harold (St. Paul’s College,
can be expressed in the form Form 6), CHIU Yik Yin (St. Joseph’s
College, Form 4), CHUNG Tat Chi
x + y − 5z , where x, y, z are integers.
2 2 2
(Queen Elizabeth School, Form 4), LAW Anglo-Chinese School, Form 5), HON
Siu Lun (Ming Kei College, Form 6), NG Chin Wing (Pui Ching Middle School,
Problem 127. For positive real numbers Cheuk Chi (Tsuen Wan Public Ho Chuen Form 6) & LEUNG Wai Ying (Queen
a, b, c with a + b + c = abc, show that Yiu Memorial College), WONG Wing Elizabeth School, Form 6).
1 1 1 3 Hong (La Salle College, Form 3) & Problem 123. Show that every convex
+ + ≤ , YEUNG Kai Sing (La Salle College,
1+ a 2
1+ b 2
1+ c 2 2 Form 4). quadrilateral with area 1 can be covered
and determine when equality occurs. by some triangle of area at most 2.
For an integer n ≥ 7, n is either of the
(Source: 1998 South Korean Math (Source: 1989 Wuhu City Math
form 2j + 1 (j > 2) or 4k(k > 1) or 4k + 2(k
Olympiad) Competition)
> 1). If n = 2j + 1, then j and j + 1 are
Solution. CHAO Khek Lun Harold (St.
Problem 128. Let M be a point on relatively prime and n = j + (j + 1). If n =
Paul’s College, Form 6), CHUNG Tat
segment AB. Let AMCD, BEHM be 4k, then 2k - 1 (>1) and 2k + 1 are Chi (Queen Elizabeth School, Form 4) &
squares on the same side of AB. Let the relatively prime and n = (2k - 1) + (2k + 1). LEUNG Wai Ying (Queen Elizabeth
If n = 4k + 2, then 2k - 1 and 2k + 3 are School, Form 6).
circumcircles of these squares intersect
at M and N. Show that B, N, C are relatively prime and n = (2k - 1) + (2k + A
collinear and H is the orthocenter of 1).
∆ABC. (Source: 1979 Henan Province Other commended solvers: HON Chin
Math Competition) Wing (Pui Ching Middle School, Form 6),
B E D
LEUNG Wai Ying (Queen Elizabeth
Problem 129. If f(x) is a polynomial of School, Form 6), NG Ka Chun
degree 2m+1 with integral coefficients Bartholomew (Queen Elizabeth School,
Form 6) & WONG Tak Wai Alan F G
for which there are 2m+1 integers C
(University of Toronto).
k1 , k 2 , , k 2 m +1 such that f ( k i ) = 1 for
i = 1, 2, , 2m + 1, prove that f(x) is not Problem 122. Prove that the product of Let ABCD be a convex quadrilateral with
the product of two nonconstant the lengths of the three angle bisectors of a area 1. Let AC meet BD at E. Without
polynomials with integral coefficients. triangle is less than the product of the loss of generality, suppose AE ≥ EC.
lengths of the three sides. (Source: 1957 Construct ∆AFG, where lines AB and
Problem 130. Prove that for each Shanghai Junior High School Math AD meet the line parallel to BD through
positive integer n, there exists a circle in Competition). C at F and G respectively. Then ∆ABE
the xy-plane which contains exactly n
Solution. YEUNG Kai Sing (La Salle is similar to ∆AFC. Now AE ≥ EC
lattice points in its interior, where a College, Form 4). implies AB ≥ BF . Let [X Y  Z] denote
lattice point is a point with integral A the area of polygon XY  Z , then [ABC]
coordinates. (Source: H. Steinhaus,
≥ [FBC]. Similarly, [ADC] ≥ [GDC].
Zadanie 498, Matematyka 10 (1957), p.
Since [ABC] + [ADC] = [ABCD] = 1,
58) F E
so [AFG] = [ABCD] + [FBC] + [GDC]
K
***************** ≤ 2[ABCD] = 2 and ∆AFG covers
Solutions ABCD.
B C
***************** D Problem 124. Find the least integer n
Mathematical Excalibur, Vol. 6, No. 2, Apr 01 – May 01 Page 4

such that among every n distinct numbers So tan 2 1$ , tan 2 3, tan 2 5$ , , tan 2 89$ comes from separating the digits and
a1 , a 2 ,  , a n , chosen from [1,1000], are the 45 roots of this equation. observing that multiplying by 10 is a
there always exist a i , a j such that Therefore, their sum is C88 90
= 4005. shifting operation in base 10. So for
0 < ai −a j < 1 + 33 ai a j . a + 2b, we take X to be the set of
positive integers whose base 2 even
(Source: 1990 Chinese Team Training
Olympiad Corner position digits from the end are 0, then
Test)
(continued from page 1) the problem is solved for positive
Solution. CHAO Khek Lun Harold (St. integers.
Paul’s College, Form 6), CHUNG Tat How many of these permutations will
How about the original problem with
Chi (Queen Elizabeth School, Form 4) & have no terms of the sequence s1 , …, integers? It is tempting to let X be the set
LEUNG Wai Ying (Queen Elizabeth s100 divisible by three? of positive or negative integers whose
School, Form 6).
base 2 even position digits from the end
Problem 3. Let A = ( a1 , a 2 ,  , a 2000 ) be
For n ≤ 10, let ai = i 3 (i = 1, 2, , n ). a sequence of integers each lying in the
are 0. It does not work as the example 1
Then the inequality cannot hold since + 2 ⋅ 1 = 3 = 5 + 2 ( -1) shows uniqueness
interval [-1000, 1000]. Suppose that the
fails. Now what other ways can we
0 < i 3 − j 3 implies i − j ≥ 1 and so entries in A sum to 1. Show that some describe the set X we used in the last
i 3 − j 3 = (i − j ) 3 + 3ij (i − j ) ≥ 1 + 3ij . nonempty subsequence of A sums to paragraph? Note it is also the set of
zero. positive integers whose base 4
For n = 11, divide [1,1000] into intervals
representations have only digits 0 or 1.
[k 3 + 1, ( k + 1) 3 ] for k = 0, 1, …, 9. By Problem 4. Let ABCD be a convex
How can we take care of uniqueness and
pigeonhole principle, among any 11 quadrilateral with
negative integers at the same time? One
distinct numbers a1 , a2 , , a11 in [1, ∠CBD = 2∠ADB , idea that comes close is the Bachet
1000], there always exist a i , a j , say ∠ABD = 2∠CDB weights.
and AB = CB. The brilliant idea in the official solution
ai > a j , in the same interval. Let
Prove that AD = CD. of the 1996 USAMO is do things in base
x = 3 ai and y = 3 a j , then 0 < x - y < 1 (−4). That is, show every integer has a
Problem 5. Suppose that the real k
∑ ci ( − 4 ) ,
i
and 0 < ai − a j = x 3 − y 3 = ( x − y ) 3 + numbers a1 , a 2 , , a100 satisfy unique representation as
i =0
3xy ( x − y ) < 1 + 3xy = 1 + 33 ai a j . a1 ≥ a 2 ≥  ≥ a100 ≥ 0, where each ci = 0, 1, 2 or 3 and ck ≠ 0.
a1 + a 2 ≤ 100 Then let X be the set of integers whose
Other commended solvers: NG Cheuk
and a 3 + a 4 +  + a100 ≤ 100. base (−4) representations have only
Chi (Tsuen Wan Public Ho Chuen Yiu
Memorial College), NG Ka Chun Determine the maximum possible value ci = 0 or 1 will solve the problem.
Bartholomew (Queen Elizabeth School, of a12 + a 22 +  + a100
2
, and find all To show that an integer n has a base (−4)
Form 6), WONG Wing Hong (La Salle
College, Form 3) & YEUNG Kai Sing possible sequences a1 , a 2 , , a100 which representation, find an integer m such that
(La Salle College, Form 4). achieve this maximum. 4 0 + 4 2 +  + 4 2 m ≥ n and express

Problem 125. Prove that n + 3 (41 + 4 3 +  + 4 2 m −1 )


2m
tan 2 1$ + tan 2 3$ + tan 2 5 $ +  + tan 2 89 $
in base 4 as ∑ bi 4 i . Now set c2i = b2i
is an integer. Base n Representations i =0

Solution. CHAO Khek Lun (St. Paul’s (continued from page 2) and c2i −1 = 3 − b2i −1 . Then
College, Form 6). 2m
Now try an example, say n = 12345. We
$ $ $ $
n = ∑ ci ( − 4 ) i .
For θ = 1 , 3 , 5 , , 89 , we have cosθ can write n in more than one ways in the i =0
≠ 0 and cos 90θ = 0 . By de Moivre’s form a + 10b. Remember we want a, b To show the uniqueness of base (−4)
theorem, cos 90θ + i sin 90θ = (cos θ + to be unique in the set X. Now for b in X,
representation of n, suppose n has two
i sin θ ) 90 . Taking the real part of both 10b will shift the digits of b to the left
distinct representations with digits ci 's
one space and fill the last digit with a 0.
sides, we get and d i 's. Let j be the smallest integer
Now we can try writing n = 12345 =
45 such that c j ≠ d j . Then
0 = ∑ ( −1) k C 290k cos 90 − 2 k θ sin 2 k θ . 10305 + 10(204). So if we take X to be
k =0 the positive integers whose even position k
0 = n − n = ∑ (ci − d i )( −4)i
Dividing by cos θ on both sides and
90 digits from the end are 0, then the i= j
letting x = tan 2 θ , we get problem will be solved for n = a + 10b.
would have a nonzero remainder when
How about n = a + 2b ? If the reader
45 divided by 4 j +1 , a contradiction.
0 = ∑ ( −1) k C 290k x k . examines the reasoning in the case
k =0 a + 10b, it is easy to see the success

Potrebbero piacerti anche